Difference between revisions of "1993 AIME Problems/Problem 15"

m
(the WOOT group did this problem. I was part of it. Somebody put the image up.)
Line 3: Line 3:
  
 
== Solution ==
 
== Solution ==
{{solution}}
+
{{image}}
 +
 
 +
From the [[Pythagorean Theorem]], <math>AH^2+CH^2=1994^2</math>, and <math>(1995-AH)^2+CH^2=1993^2</math>. Subtracting those two equations yields <math>AH^2-(1995-AH)^2=3987</math>. After simplification, we see that <math>2*1995AH-1995^2=3987</math>, or <math>AH=\frac{1995}{2}+\frac{3987}{2*1995}</math>. Note that <math>AH+BH=1995</math>. Therefore we have that <math>BH=\frac{1995}{2}-\frac{3987}{2*1995}</math>. Therefore <math>AH-BH=\frac{3987}{1995}</math>.
 +
 
 +
Now note that <math>RS=|HR-HS|</math>, <math>RH=\frac{BH+CH-BC}{2}</math>, and <math>HS=\frac{CH+AH-AC}{2}</math>. Therefore we have
 +
 
 +
<math>RS=|\frac{BH+CH-BC-CH-AH+AC}{2}|=\frac{|BH-AH-1993+1994|}{2}</math>.
 +
 
 +
Plugging in <math>AH-BH</math> and simplifying, we have <math>RS=\frac{1992}{1995*2}=\frac{332}{665}</math>.
  
 
== See also ==
 
== See also ==
 
{{AIME box|year=1993|num-b=14|after=Last question}}
 
{{AIME box|year=1993|num-b=14|after=Last question}}
 +
[[Category:Intermediate Geometry Problems]]

Revision as of 21:27, 3 January 2009

Problem

Let $\overline{CH}$ be an altitude of $\triangle ABC$. Let $R\,$ and $S\,$ be the points where the circles inscribed in the triangles $ACH\,$ and $BCH^{}_{}$ are tangent to $\overline{CH}$. If $AB = 1995\,$, $AC = 1994\,$, and $BC = 1993\,$, then $RS\,$ can be expressed as $m/n\,$, where $m\,$ and $n\,$ are relatively prime integers. Find $m + n\,$.

Solution


An image is supposed to go here. You can help us out by creating one and editing it in. Thanks.


From the Pythagorean Theorem, $AH^2+CH^2=1994^2$, and $(1995-AH)^2+CH^2=1993^2$. Subtracting those two equations yields $AH^2-(1995-AH)^2=3987$. After simplification, we see that $2*1995AH-1995^2=3987$, or $AH=\frac{1995}{2}+\frac{3987}{2*1995}$. Note that $AH+BH=1995$. Therefore we have that $BH=\frac{1995}{2}-\frac{3987}{2*1995}$. Therefore $AH-BH=\frac{3987}{1995}$.

Now note that $RS=|HR-HS|$, $RH=\frac{BH+CH-BC}{2}$, and $HS=\frac{CH+AH-AC}{2}$. Therefore we have

$RS=|\frac{BH+CH-BC-CH-AH+AC}{2}|=\frac{|BH-AH-1993+1994|}{2}$.

Plugging in $AH-BH$ and simplifying, we have $RS=\frac{1992}{1995*2}=\frac{332}{665}$.

See also

1993 AIME (ProblemsAnswer KeyResources)
Preceded by
Problem 14
Followed by
Last question
1 2 3 4 5 6 7 8 9 10 11 12 13 14 15
All AIME Problems and Solutions